Proving "Rotation Matrix is Orthogonal: Necessary & Sufficient

In summary, a transformation is orthogonal if and only if it is a length-preserving transformation, meaning that it preserves the length of vectors, and it preserves angles between vectors.
  • #1
brotherbobby
603
150
I'd like to prove the fact that - since a rotation of axes is a length-preserving transformation, the rotation matrix must be orthogonal.

By the way, the converse of the statement is true also. Meaning, if a transformation is orthogonal, it must be length preserving, and I have been able to prove it. This is the so called "sufficient" statement.

But how to prove the "necessary" statement above? Any help?
 
Physics news on Phys.org
  • #2
I am sure there are easier ways to do this than this procedure. Take u to be a vector. Take v to be the transformed vector, of the same length. Translate v to have the same origin as u. Now you can find the angle between u and v (with the same origin) by taking the dot product. The cross product between u and v (with the same origin) will provide a vector perpendicular to both u and v, call it w. Vector w will have length sin (angle between u and v). Divide the w by sin angle to make the vector unit length. Now you can form the rotation matrix that will rotate u into v, or vice versa, v into u. You can multiply either by the transpose to verify you come up with the unit matrix.

You might try this as a last resort although I am sure there are easier ways. If you do accomplish this, you will learn a lot about rotations.
 
  • #3
brotherbobby said:
if a transformation is orthogonal, it must be length preserving, and I have been able to prove it.

How did you prove it? If all the steps in your proof can be reversed, then you have the proof you are looking for.
 
  • #4
PeterDonis said:
How did you prove it? If all the steps in your proof can be reversed, then you have the proof you are looking for.
No am afraid, it is not like that, the argument cannot be reversed like that. Let me present it so that you know what I mean.

Imagine the rotation matrix to be orthogonal and prove that it will preserve the length of a vector when the coordinate system undergoes rotation.

The "new" length of the vector is ##x'_i x'_i = R_{ij}x_j R_{ik} x_k \text{, (summation on repeated indices implied)} = R_{ij} R_{ik} x_j x_k = \delta_{jk} x_j x_k \text{(since the rotation matrix is orthogonal)}=x_k x_k = x_i x_i ##,

which shows that the length is invariant.

The argument cannot be reversed. I need to begin from the fact that the length is an invariant, i.e. ##x'_i x'_i = x_i x_i ## and then derive that if it is so, the rotation matrix has to be orthogonal, i.e. ##R_{ij} R_{ik}=\delta_{jk}##. Any help?
 
  • #5
brotherbobby said:
No am afraid, it is not like that, the argument cannot be reversed like that. ... Any help?

Yes, you can reverse your argument. [tex]x_{i} x_{i} = \delta_{jk} x_{j}x_{k} , \ \ \ \ \ \ \ (1)[/tex] [tex]\bar{x}_{i} \bar{x}_{i} = R_{ij}R_{ik} x_{j} x_{k} . \ \ \ \ \ (2)[/tex] So, if the left-hand-sides of (1) and (2) are equal, then [itex]R_{ij}R_{ik} = \delta_{jk}[/itex]. You can also do it by writing [tex]R_{ij} = (e^{\omega})_{ij} = \delta_{ij} + \omega_{ij} + O(\omega^{2}) ,[/tex] work to first order in [itex]\omega[/itex] and show that [itex]\omega_{ij} = - \omega_{ji}[/itex]. This means that the matrix [itex]R[/itex] is orthogonal: [tex]R^{-1}_{ij} = R_{ji} = R^{T}_{ij} .[/tex] More generally, given [itex]\bar{x} = Rx[/itex], where [itex]x = (x_{1}, x_{2}, \cdots , x_{n})^{T}[/itex] and [itex]R[/itex] is a real [itex]n \times n[/itex], you can easily show that [tex]\{ (\bar{x})^{2} = (x)^{2} \} \ \Leftrightarrow \ \{ R^{-1} = R^{T} \} .[/tex]
 
Last edited:
  • #6
Definition: Let X be an inner product space over ##\mathbb R##. For each ##x,y\in X-\{0\}##, define the angle between ##x## and ##y##, denoted by ##\theta(x,y)##, by
$$\cos\theta(x,y)=\frac{\langle x,y\rangle}{\|x\|\|y\|}.$$
Note that the Cauchy-Schwartz inequality ensures that the right-hand side is in the interval [-1,1].

Theorem: Let X be an inner product space over ##\mathbb R##. If ##R:X\to X## is surjective onto ##X## and preserves distances and angles, then ##R## is an orthogonal linear transformation (i.e. a linear transformation that preserves norms).

Proof: We will prove that ##R## preserves norms. Let ##x\in X##. Since ##R## preserves distances, we have ##\|Rx\|=\|R(x-0)\|=\|x-0\|=\|x\|##. Since ##x## is an arbitrary element of ##X##, this means that ##R## preserves norms.

We will prove that ##R## preserves inner products. Let ##x,y\in X-\{0\}##. Since ##R## preserves angles and norms, we have
$$\frac{\langle x,y\rangle}{\|x\|\|y\|}=\cos\theta(x,y)=\cos\theta(Rx,Ry)=\frac{\langle Rx,Ry\rangle}{\|Rx\|\|Ry\|} =\frac{\langle Rx,Ry\rangle}{\|x\|\|y\|}.$$ This implies that ##\langle x,y\rangle=\langle Rx,Ry\rangle##. Since ##x,y## are arbitrary elements of ##X##, this means that ##R## preserves inner products.

We will prove that ##R## is linear. Let ##a,b\in\mathbb R##. Let ##x,y,z\in X##. Let ##w\in X## be such that ##Rw=z##.
\begin{align*}
&\langle z,R(ax+by)\rangle =\langle Rw,R(ax+by)\rangle =\langle w,ax+by\rangle =a\langle w,x\rangle+b\langle w,y\rangle =a\langle Rw,Rx\rangle+b\langle Rw,Ry\rangle\\
& =\langle z,aRx+bRy\rangle.
\end{align*} Since ##z## is an arbitrary element of ##X##, this implies that ##R(ax+by)=aRx+bRy##. Since ##a,b## are arbitrary real numbers and ##x,y## are arbitrary elements of ##X##, this means that ##R## is linear.

Comment: You may still be wondering why a norm-preserving linear transformation would be orthogonal in the sense ##R^TR=I##. The standard inner product on ##\mathbb R^3## is given by ##\langle x,y\rangle =x^Ty##. So for all ##x\in\mathbb R^3##, we have
$$x^Tx=\langle x,x\rangle =\|x\|^2=\|Rx\|^2=\langle Rx,Rx\rangle =(Rx)^T(Rx)=x^TR^TRx.$$ This implies that ##R^TR=I##. (The idea is to make clever choices of x that I don't have time to explain right now).
 
Last edited:

1. What is a rotation matrix?

A rotation matrix is a mathematical tool used to represent a rotation transformation in three-dimensional space. It is a square matrix with dimensions of 3x3 and has special properties that make it useful for performing rotations.

2. What does it mean for a rotation matrix to be orthogonal?

An orthogonal matrix is a matrix whose columns (and rows) are all perpendicular to each other. In the context of rotation matrices, this means that the matrix represents a rotation that preserves the length and angle of vectors. In other words, it is a rigid, non-deforming rotation.

3. Why is it necessary for a rotation matrix to be orthogonal?

A rotation matrix must be orthogonal in order to accurately represent a rotation transformation. If a matrix is not orthogonal, it will not preserve the length and angle of vectors, resulting in a distorted rotation. Additionally, orthogonal matrices have nice properties that make them easier to work with in mathematical calculations.

4. How do you prove that a rotation matrix is orthogonal?

To prove that a matrix is orthogonal, you must show that its columns (and rows) are all perpendicular to each other and have a length of 1. This can be done using various mathematical methods, such as using the dot product or matrix multiplication.

5. What is the relationship between orthogonality and determinants in a rotation matrix?

In a rotation matrix, the determinant is always equal to 1. This is because the determinant represents the scaling factor of the matrix, and for an orthogonal matrix, the scaling factor must be 1 in order to preserve the length of vectors. In other words, the determinant being equal to 1 is a necessary and sufficient condition for a rotation matrix to be orthogonal.

Similar threads

  • Special and General Relativity
Replies
22
Views
2K
Replies
12
Views
939
  • Topology and Analysis
Replies
3
Views
162
  • Special and General Relativity
Replies
8
Views
2K
Replies
7
Views
892
Replies
7
Views
1K
  • Advanced Physics Homework Help
Replies
4
Views
859
Replies
40
Views
2K
  • Calculus and Beyond Homework Help
Replies
3
Views
1K
Replies
1
Views
908
Back
Top